Question Video: Determining Current Directions from Net Magnetic Field | Nagwa Question Video: Determining Current Directions from Net Magnetic Field | Nagwa

Question Video: Determining Current Directions from Net Magnetic Field Physics

Several horizontal pairs of parallel conducting wires are stacked vertically. The magnitude of the current in each wire is the same. A cross section of the resultant magnetic field due to the currents is shown in the diagram. Which of the configurations of current direction shown would produce the resultant magnetic field?

08:38

Video Transcript

Several horizontal pairs of parallel conducting wires are stacked vertically. The magnitude of the current in each wire is the same. A cross section of the resultant magnetic field due to the currents is shown in the diagram. Which of the configurations of current direction shown would produce the resultant magnetic field? (A) I, (B) III, (C) I and III, (D) I and IV, (E) III and IV.

In our diagram, we see this arrangement of vertically stacked pairs of conducting wires. This is one pair here, and that pair is stacked on top of this pair, which is stacked on top of this pair, and so on. Each one of the wires in these pairs carries current either into or out of the screen. Those directions are not indicated in the diagram. What we do see though is a cross section of the overall magnetic field formed by all these current-carrying wires. Specifically, we see magnetic field lines of this field.

Based on the appearance of these field lines, we want to be able to choose which of our four answer options shows us current direction that would generate a magnetic field that looks like this. To begin seeing how this can work, let’s clear some space at the top of our screen and remind ourselves about the magnetic fields that are produced by current-carrying wires.

Here, we imagine a wire carrying current into the screen. That’s one possibility for the wires in our diagram. And here, we have a wire carrying current out of the screen. Generally speaking, given a straight wire carrying a current 𝐼, then if we point the thumb on our right hand in the direction of that current, the fingers of that hand will be able to curl closed in the direction of the magnetic field around the wire. For this wire then, the magnetic field would point in these directions.

Applying the same right-hand rule to this wire carrying current into the screen, if we point the thumb on our right hand in that same direction, then the fingers on our right hand can curl closed in this direction, clockwise around the wire. This then is the magnetic field direction around such a current. If we then consider the magnetic field generated by a wire carrying current out of the screen towards us, pointing our right thumb in that direction allows our fingers to curl closed this way, counterclockwise.

Even though for each one of these currents we’ve only drawn one closed field line, it’s worth noting that the magnetic field from either one of these currents extends out infinitely far away from the axis of the wire. This tells us that the magnetic field, say, here right between the two wires is a combination of the magnetic field generated by this wire and the magnetic field generated by this one.

Since the wire on the left generates a clockwise-pointing magnetic field, at the location we’ve picked that field will point straight downward. For the other current, the one that points counterclockwise around the wire, note that at this location we’ve picked that field will also point in the same direction straight down.

Magnetic field is a vector quantity; it has magnitude and direction. If we add together these two individual magnetic field vectors, then we would get a vector that looks something like this. This represents the overall or the net magnetic field midway between these wires.

We’re seeing then that given this pair of current-carrying wires, the magnetic field between them will not be zero, but rather it will be quite strong. On our diagram, areas of relatively greater magnetic field strength are indicated by greater density of magnetic field lines. Where there are lots of field lines packed closely together, the field is relatively strong.

As a next step, let’s try stacking an identical pair of current-carrying wires right below this one. Given another pair of wires like this, the magnetic field lines around each one individually will point the same way as before. And so if we consider the magnetic field that’s formed between this particular pair of current-carrying wires, just like for the pair above the net magnetic field will point strongly downward.

We can see then that as we stack pairs of current-carrying wires like this one on top of another, what we get is a magnetic field in between them with compounding strength. That is, the magnetic fields formed by each individual pair add to the magnetic fields formed by the other pairs. If our diagram consisted of current-carrying wires like these pairs stacked one on top of another, then we would expect a large number of long, straight magnetic field lines running right through the centers of the pairs. But what we see instead is that the magnetic field lines seem to form loops around each current-carrying wire individually.

We can say then that the current directions in our configuration do not look like this pattern repeated over and over. Notice that this is the current direction configuration shown in answer option III. Because the field lines we see in our diagram do not match the field lines in between the pairs of current-carrying wires we see for the configuration demonstrated in answer option III, we can eliminate this choice from consideration.

To test out a different configuration of current-carrying wires, let’s say that we reverse the directions of the current in each of the wires in this pair. That is, we’ll make the current in the wire on the left come out of the screen. Our right-hand rule shows us that would create a magnetic field like this. And we’ll let the current in the other wire point now into the screen. This would yield a magnetic field pointing clockwise.

Considering now the magnetic fields only from this pair of wires, let’s now see what the net magnetic field is at a point midway between them. The wire on the left, with a counterclockwise magnetic field, would create a magnetic field at this point pointing upward. And likewise, the magnetic field from the current on the right at this location points in the same direction. If we add these magnetic fields together, notice that we get a relatively stronger magnetic field pointing upward. What we’re seeing is that the magnetic field created by this pair of wires competes with the magnetic field created by this pair above it, the overall effect being that the magnetic fields caused by these stacked pairs of wires would more or less separate out along this dashed line.

If we look back at our diagram, we can see this happening there. The magnetic fields created by these pairs of stacked wires are effectively compartmentalized to the region of that pair. This tells us that if we were to continue this alternating pattern of current-carrying pairs of wires — that is, if the next pair in the stack carried currents like this and then the pair after that like this — the overall effect would be to create a magnetic field that is demonstrated in our diagram, that is, a field that is compartmentalized according to each pair.

If we look for this pattern of current-carrying wires among our answer options, we see that there’s a match with answer choice I. Therefore, whatever letter we end up choosing to represent our final answer must include this answer choice. This means we can eliminate option (B), which doesn’t include I as well as option (E), which also doesn’t include this configuration. Along with this, we can eliminate answer choice (C). We could have done this earlier because this choice includes answer option III.

The remaining answers raise the question of whether configuration IV over here could also create the magnetic field we see in our diagram. An important thing to notice about this field is that we don’t know its direction at any point. In our sketches, we’ve drawn arrowheads to indicate direction, but in our diagram, we don’t see any directional indication. Given a closed loop of a magnetic field line, say, this one right here, we don’t know then whether this loop points clockwise or counterclockwise. This means that for all of our current-carrying pairs, if we were to swap the left-side current direction with the right-side current direction — that is, if the current directions would look like this — then we would once again get an overall magnetic field that was effectively segmented or compartmentalized by each pair.

The directions of the field lines in each case will reverse, but recall that those directions aren’t indicated here. And the pattern overall of the field lines would remain the same. All this is to say that answer choice IV is a legitimate configuration of current directions. Whether the current directions in our wires look like this or like this, we would still get a magnetic field that looks like the one we see in our diagram. Our final answer then we’ll be option (D).

Before finishing up though, let’s consider why configuration II is not a valid choice. If all of the wires in our configuration carried current pointing out of the screen, then the magnetic field around each one would point counterclockwise. If we consider then the magnetic field formed midway between one of these pairs, we see that due to the current on the left there is an upward pointing magnetic field, while due to the current on the right there is an equal and oppositely directed magnetic field. These fields will combine to give a net magnetic field of effectively zero. And the same thing will happen for every other pair in our stack.

If our diagram matched this configuration, we would expect to see almost no magnetic field lines in between the pairs of wires. Because we do though, we can know that configuration II is not a representation of the currents in these wires. Rather, it’s configurations I and IV only that demonstrate current directions that would generate the magnetic field shown.

Join Nagwa Classes

Attend live sessions on Nagwa Classes to boost your learning with guidance and advice from an expert teacher!

  • Interactive Sessions
  • Chat & Messaging
  • Realistic Exam Questions

Nagwa uses cookies to ensure you get the best experience on our website. Learn more about our Privacy Policy